Added topic tags to some S&J-4 problems + minor typos
[course.git] / latex / problems / Serway_and_Jewett_4 / problem24.07.tex
index 4112ccb949c191d4421f854084034cdedf1fd04d..0b3e684b272be0413edcf6bfbb8e0935e8ca6fc7 100644 (file)
@@ -1,4 +1,4 @@
-\begin{problem*}{24.7}
+\begin{problem*}{24.7} % EM waves
 Figure 24.3 shows a plane electromagnetic sinosoidal wave propogating
 in the $x$ direction.  Suppose the wavelength is $50.0\U{m}$ and the
 electric field vibrates in the $xy$ plane with an amplitude of
@@ -11,7 +11,7 @@ its magnidude in the form
 \begin{equation}
  B = B_\text{max}\cos(kx-\omega t)
 \end{equation}
-\end{problem*} % problem 24.7
+\end{problem*}
 
 \begin{solution}
 \Part{a}